Difference between revisions of "1996 AHSME Problems/Problem 21"

 
(3 intermediate revisions by 2 users not shown)
Line 21: Line 21:
 
<math> \text{(A)}\ 115^\circ\qquad\text{(B)}\ 120^\circ\qquad\text{(C)}\ 130^\circ\qquad\text{(D)}\ 135^\circ\qquad\text{(E)}\ \text{not uniquely determined} </math>
 
<math> \text{(A)}\ 115^\circ\qquad\text{(B)}\ 120^\circ\qquad\text{(C)}\ 130^\circ\qquad\text{(D)}\ 135^\circ\qquad\text{(E)}\ \text{not uniquely determined} </math>
  
==Solution==
+
==Solution 1==
 +
 
 +
Redraw the figure as a concave pentagon <math>ADECB</math>:
 +
 
 +
<asy>
 +
size(120);
 +
pair B=origin, A=1*dir(70), M=foot(A, B, (3,0)), C=reflect(A, M)*B, E=foot(B, A, C), D=1*dir(20);
 +
dot(A^^B^^C^^D^^E);
 +
draw(A--D--E--C--B--A);
 +
markscalefactor=0.005;
 +
draw(rightanglemark(D, E, C));
 +
dot(A^^B^^C^^D^^E);
 +
pair point=midpoint(A--M);
 +
label("$A$", A, dir(point--A));
 +
label("$B$", B, dir(point--B));
 +
label("$C$", C, dir(point--C));
 +
label("$D$", D, dir(point--D));
 +
label("$E$", E, dir(point--E));
 +
</asy>
 +
 
 +
The angles of the pentagon will still sum to <math>180^\circ \cdot 3 = 540^\circ</math>, regardless of whether the pentagon is concave or not.  As a quick proof, note that the nine angles of three original triangles <math>\triangle  AEB</math>, <math>\triangle CBE</math>, and <math>\triangle DEA</math> all make up the angles of the pentagon without overlap.
 +
 
 +
Since reflex <math>\angle E = 270^\circ</math>, we have:
 +
 
 +
<math>\angle C + \angle B + \angle A + \angle D = 540^\circ - 270^\circ = 270^\circ</math>.
 +
 
 +
From isosceles <math>\triangle ABC</math>, we get <math>\angle B = \angle C</math>, so:
 +
 
 +
<math>2\angle C + \angle D + \angle A = 270^\circ</math>
 +
 
 +
From isosceles <math>\triangle ABD</math>, we get <math>\angle A = \angle D</math>, so:
 +
 
 +
<math>2\angle C + 2\angle D = 270^\circ</math>
 +
 
 +
<math>\angle C + \angle D = 135^\circ</math>, which is answer <math>\boxed{D}</math>
 +
 
 +
==Solution 2==
 +
 
 +
Let <math>m\angle ABC = x</math>. By the isosceles triangle theorem, we have <math>m\angle ACB = x</math> and <math>m\angle BAD = m\angle BDA</math>. Because the angles of a triangle sum to <math>\pi</math>, we have <math>m\angle BAC = \pi - 2x</math>, then <math>m\angle ABD=\frac{\pi}{2}-(\pi-2x)=2x-\frac{\pi}{2}</math>. Then we have <math>m\angle BAD + m\angle BDA = \pi - m\angle ABD</math>. Substituting, this becomes <math>2m\angle BDA = \pi-\left(\frac{\pi}{2}-(\pi-2x)\right) \to m \angle BDA = \frac{3 \pi}{4} - x  </math>. Adding <math>x</math>, which is <math>m\angle ACB</math>, we have <math>m\angle BDA + m\angle ACB = \frac{3 \pi}{4} = 135^\circ \to \boxed{\textbf{D}}</math>
  
  
 
==See also==
 
==See also==
 
{{AHSME box|year=1996|num-b=20|num-a=22}}
 
{{AHSME box|year=1996|num-b=20|num-a=22}}
 +
{{MAA Notice}}

Latest revision as of 21:40, 13 January 2014

Problem

Triangles $ABC$ and $ABD$ are isosceles with $AB=AC=BD$, and $BD$ intersects $AC$ at $E$. If $BD$ is perpendicular to $AC$, then $\angle C+\angle D$ is

[asy] size(120); pair B=origin, A=1*dir(70), M=foot(A, B, (3,0)), C=reflect(A, M)*B, E=foot(B, A, C), D=1*dir(20); dot(A^^B^^C^^D^^E); draw(A--D--B--A--C--B); markscalefactor=0.005; draw(rightanglemark(A, E, B)); dot(A^^B^^C^^D^^E); pair point=midpoint(A--M); label("$A$", A, dir(point--A)); label("$B$", B, dir(point--B)); label("$C$", C, dir(point--C)); label("$D$", D, dir(point--D)); label("$E$", E, dir(point--E)); [/asy]

$\text{(A)}\ 115^\circ\qquad\text{(B)}\ 120^\circ\qquad\text{(C)}\ 130^\circ\qquad\text{(D)}\ 135^\circ\qquad\text{(E)}\ \text{not uniquely determined}$

Solution 1

Redraw the figure as a concave pentagon $ADECB$:

[asy] size(120); pair B=origin, A=1*dir(70), M=foot(A, B, (3,0)), C=reflect(A, M)*B, E=foot(B, A, C), D=1*dir(20); dot(A^^B^^C^^D^^E); draw(A--D--E--C--B--A); markscalefactor=0.005; draw(rightanglemark(D, E, C)); dot(A^^B^^C^^D^^E); pair point=midpoint(A--M); label("$A$", A, dir(point--A)); label("$B$", B, dir(point--B)); label("$C$", C, dir(point--C)); label("$D$", D, dir(point--D)); label("$E$", E, dir(point--E)); [/asy]

The angles of the pentagon will still sum to $180^\circ \cdot 3 = 540^\circ$, regardless of whether the pentagon is concave or not. As a quick proof, note that the nine angles of three original triangles $\triangle  AEB$, $\triangle CBE$, and $\triangle DEA$ all make up the angles of the pentagon without overlap.

Since reflex $\angle E = 270^\circ$, we have:

$\angle C + \angle B + \angle A + \angle D = 540^\circ - 270^\circ = 270^\circ$.

From isosceles $\triangle ABC$, we get $\angle B = \angle C$, so:

$2\angle C + \angle D + \angle A = 270^\circ$

From isosceles $\triangle ABD$, we get $\angle A = \angle D$, so:

$2\angle C + 2\angle D = 270^\circ$

$\angle C + \angle D = 135^\circ$, which is answer $\boxed{D}$

Solution 2

Let $m\angle ABC = x$. By the isosceles triangle theorem, we have $m\angle ACB = x$ and $m\angle BAD = m\angle BDA$. Because the angles of a triangle sum to $\pi$, we have $m\angle BAC = \pi - 2x$, then $m\angle ABD=\frac{\pi}{2}-(\pi-2x)=2x-\frac{\pi}{2}$. Then we have $m\angle BAD + m\angle BDA = \pi - m\angle ABD$. Substituting, this becomes $2m\angle BDA = \pi-\left(\frac{\pi}{2}-(\pi-2x)\right) \to m \angle BDA = \frac{3 \pi}{4} - x$. Adding $x$, which is $m\angle ACB$, we have $m\angle BDA + m\angle ACB = \frac{3 \pi}{4} = 135^\circ \to \boxed{\textbf{D}}$


See also

1996 AHSME (ProblemsAnswer KeyResources)
Preceded by
Problem 20
Followed by
Problem 22
1 2 3 4 5 6 7 8 9 10 11 12 13 14 15 16 17 18 19 20 21 22 23 24 25 26 27 28 29 30
All AHSME Problems and Solutions

The problems on this page are copyrighted by the Mathematical Association of America's American Mathematics Competitions. AMC logo.png